Tải bản đầy đủ (.pdf) (25 trang)

NGHIỆM và TÍNH CHẤT NGHIỆM của đa THỨC sử DỤNG số PHÚC để GIẢI bài TOÁN về đa THỨC

Bạn đang xem bản rút gọn của tài liệu. Xem và tải ngay bản đầy đủ của tài liệu tại đây (209.15 KB, 25 trang )

CHUYÊN ĐỀ
SỬ DỤNG SỐ PHỨC GIẢI CÁC TOÁN VỀ ĐA THỨC
Phạm Xuân Thịnh, trường THPT chuyên Hạ Long, Quảng Ninh

1

I. MỞ ĐẦU
0.1

Lí do chọn đề tài

Đa thức là một đối tượng được nghiên cứu nhiều nhất trong Toán học. Ngay từ
THCS, THPT đa thức đã được đề cập rất nhiều trong chương trình sách giáo khoa
và sách chuyên khảo. Ở những bậc học này, đa thức thường được nghiên cứu về các
tính chất Đại số, tính chất Số học và tính chất Giải tích. Đến bậc Đại học và sau
Đại học, thì đa thức là một đối tượng cơ bản được nghiên cứu trong nhiều ngành
Toán học khác nhau như Đại số, Lý thuyết số, Hình học, Giải tích, Toán rời rạc,...
Có lẽ do tính quan trọng của đa thức trong Toán học mà các bài toán về đa thức
thường xuất hiện nhiều trong các kì thi học sinh giỏi, nhất là thi học sinh giỏi Quốc
gia và Quốc tế với mức độ thường được đánh giá là khó. Cái khó của các bài toán
về đa thức nhiều khi là ta không biết bắt đầu từ đâu.
Một vấn đề quan trọng đối với đa thức là vấn đề nghiệm của nó. Nhiều bài toán về
đa thức được giải quyết dựa vào việc xử lí nghiệm của nó. Tuy nhiên trên trường số
thực, một đa thức có thể có nghiệm nhưng cũng có thể vô nghiệm. Hơn nữa, thật
khó để có thể biết một đa thức có nghiệm thực hay không. Vì thế việc giải quyết
các bài toán đa thức thông qua nghiệm thực của nó sẽ trở nên vô cùng khó khăn.
Nhưng trên trường số phức lại khác, vấn đề tồn tại nghiệm của đa thức không còn
là sự trở ngại nữa. Bởi vì mọi đa thức khác hằng đều có nghiệm phức và do đó nếu
ta xử lí bài toán thông qua các nghiệm phức của nó sẽ rất thuận lợi.
Vì những lí do trên, chúng tôi quyết định chọn đề tài:”SỬ DỤNG SỐ PHỨC ĐỂ
1



Các trao đổi với tác giả về bài viết này có thể liên hệ qua email :


2
GIẢI CÁC BÀI TOÁN VỀ ĐA THỨC” nhằm nghiên cứu sâu hơn về đa thức cũng
như củng cố thêm kiến thức về số phức, phục vụ cho việc dạy và học. Với đề tài
này, chúng tôi hi vọng phần nào chia sẻ và giúp các bạn có thêm cách tiếp cận các
bài toán về đa thức. Ngoài ra, thông qua đề tài này, chúng tôi rất mong muốn nhận
được những ý kiến đánh giá, những góp ý trao đổi của các đồng nghiệp, các em học
sinh nói riêng và tất cả những ai quan tâm nói chung.

0.2

Mục đích và nhiệm vụ nghiên cứu

Dựa trên thực tế giảng dạy và học tập, mục đích và nhiệm vụ nghiên cứu của đề tài
là:
i) Nghiên cứu những tính chất sâu hơn về đa thức.
ii) Đề xuất một số biện pháp rèn luyện tư duy, năng lực giải toán và kĩ năng vận
dụng số phức để giải các bài toán về đa thức như: chia đa thức, bài toán xác
định đa thức, bài toán về đa thức bất khả quy, ...
iii) Tạo tiền đề cho những nghiên cứu về sử dụng số phức và đa thức vào các bài
toán tổ hợp.

0.3

Đối tượng học sinh

Chuyên đề dùng để dạy đối tượng học sinh chuyên toán của trường THPT, đặc biệt

là bồi dưỡng học sinh trong đội tuyển thi học sinh giỏi Quốc gia và Quốc tế.
Ký hiệu và quy ước
N : tập các số tự nhiên
Z : tập các số nguyên
Q : tập các số hữu tỉ
R : tập các số thực
C : tập các số phức
Q+ (Q− ) : tập các số hữu tỉ không âm (không dương)
Q∗ : tập các số hữu tỉ khác không

vp (a) : số mũ cao nhất của số nguyên tố p trong biểu diễn chính tắc của số nguyên a.


3

II.NỘI DUNG
1

Kiến thức chuẩn bị

Trong mục này chúng tôi nêu một số kiến thức về đa thức (không chứng minh)
nhằm phục vụ cho các nội dung của chuyên đề.
Giả sử K là một tập con khác rỗng của R. Kí hiệu K[x] là tập các đa thức với
hệ số thuộc K.
Định lý 1. Mọi đa thức bậc n (n ≥ 0) với hệ số thực đều có tối đa n nghiệm thực
(tính cả bội).
Định lý 2. Mọi đa thức bậc n (n ≥ 1) đều có đúng n nghiệm phức (tính cả bội).
Định lý 3 (Định lí Bezout). Số thực a là nghiệm của đa thức P (x) ∈ R[x] khi và
chỉ khi P (x) chia hết cho x − a, hay nói khác đi, tồn tại đa thức Q(x) ∈ R[x] sao
cho P (x) = (x − a)Q(x), ∀x ∈ R.

Định nghĩa 1. Đa thức P (x) ∈ K[x] được gọi là bất khả quy trên K nếu nó không
thể phân tích thành tích của 2 đa thức khác hằng thuộc K[x].
Định lý 4. Cho P (x) ∈ Z[x]. Khi đó, P (x) bất khả quy trên Z khi và chỉ khi P (x)
bất khả quy trên Q.
Định lý 5. Nếu các biểu thức đối xứng cơ bản của các biến x1 , x2 , . . . , xn đều nhận
giá trị nguyên (hữu tỉ), thì mọi đa thức đối xứng với hệ số nguyên của các biến
x1 , x2 , . . . , xn cũng nhận giá trị nguyên (tương ứng: hữu tỉ).

2
2.1

Sử dụng số phức giải các bài toán về đa thức
Bài toán xác định đa thức, chia hết

Ví dụ 1. Tìm tất cả các đa thức P (x) ∈ R[x] thỏa mãn điều kiện P (x).P (2x2 ) =
P (2x3 + x), ∀x ∈ R.


4
Giải. Nếu P (x) ≡ c là một đa thức hằng, thì từ giả thiết, ta có c2 = c hay c = 0
và c = 1. Xét P (x) là một đa thức khác hằng. Nếu P (x) có nghiệm thực α, thì
2α3 + α cũng là nghiệm của P (x). Xảy ra 2 trường hợp sau
• Nếu α = 0, thì P (0) = 0. Đặt P (x) = xk .Q(x) với k ≥ 1, Q(0) = 0. Khi đó,
2k .x3k Q(x)Q(2x2 ) = (2x3 + x)k Q(2x3 + x)
hay là
2k .x2k Q(x)Q(2x2 ) = (2x2 + 1)k Q(2x3 + x).
Từ đây suy ra Q(0) = 0. Mâu thuẫn !
• Nếu α = 0, thì ta xét dãy số (un ) xác định bởi u1 = α, un+1 = 2u3n +un , n ≥ 1.
Khi đó, P (un ) = 0, ∀n ≥ 1. Hơn nữa, dễ dàng thấy rằng dãy này tăng khi
α > 0 và giảm khi α < 0. Điều đó chứng tở P (x) ≡ 0. Mâu thuẫn !

Vậy P (x) không có nghiệm thực. Do đó, deg(P (x)) = 2n, n ∈ N∗ . Vì P (0) = 1 nên
tích tất cả các nghiệm của P (x) bằng 1, theo đó tích môđun của tất cả các nghiệm
cũng bằng 1. Ta sẽ chứng minh tất cả các nghiệm của P (x) đều có môđun bằng 1.
Thật vậy, nếu điều đó không đúng, gọi α là nghiệm phức có môđun lớn nhất trong
các nghiệm của P (x), thì |α| > 1. Khi ấy, từ
|2α2 + 1|2 = (2α2 + 1)(2α2 + 1) = 4|α|4 + 2(α2 + α2 ) + 1
= 4|α|2(|α|2 − 1) + 2(α + α)2 + 1 > 1,

suy ra |2α3 +α| = |α|.|2α2 +1| > |α|. Mà 2α3 +α cũng là nghiệm của P (x). Điều này
mâu thuẫn với cách chọn α. Bây giờ từ |α| = |2α3 + α| = 1, ta suy ra |2α2 + 1| = 1
hay (α + α)2 = 0, tức là α = −α. Do đó, α là số thuần ảo hay α = k.i, k ∈ R. Từ
|α| = 1 ta có |k| = 1 hay k = ±1. Thành thử, α = ±i. Theo đó, P (x) chỉ có nghiệm
là ±i và do P (x) là đa thức với hệ số thực nên bội của i và −i phải bằng nhau. Vì
thế,
P (x) = (x + i)n (x − i)n = (x2 + 1)n .
Ví dụ 2. Tìm tất cả các đa thức P (x) ∈ R[x] khác hằng thỏa mãn P (x)P (x + 1) =
P (x2 + 1), ∀x ∈ R.
Giải. Trước hết ta thấy P (x) không có nghiệm thực. Thật vậy nếu P (x) có một
nghiệm thực là a, thì a2 + 1, (a2 + 1)2 + 1, ... cũng là nghiệm. Vì
a < a2 + 1 < (a2 + 1)2 + 1 < ...


5
nên P (x) có vô số nghiệm. Điều này là vô lí, vì P (x) là đa thức khác hằng. Do đó,
deg(P (x)) = 2n, n ∈ N∗ . Gọi α là một nghiệm phức bất kì của P (x), thì dễ thấy
n
α2 + 1, α4 + 1, ..., α2 + 1, ... cũng là nghiệm của P (x). Vì P (x) là đa thức khác hằng
phải tồn tại 2 số nguyên dương m, n, m < n sao cho
n


m

m

α2 + 1 = α2 + 1 ⇐⇒ α2

n −2m

α2

− 1 = 0.

Do α = 0 nên α2 −2 = 1 hay |α| = 1. Như vậy ta chứng minh được mọi nghiệm của
P (x) đều có môđun bằng 1. Bây giờ nếu gọi α là một nghiệm của P (x), thì α2 + 1
và α2 − 2α + 2 cũng là nghiệm. Do đó,
n

m

|α| = α2 + 1 = α2 − 2α + 2 = 1.
Chú ý là
α2 + 1

2

= |α|4 + (α + α)2 − 2 |α|2 + 1 = (α + α)2

nên từ |α2 + 1| = 1 ta suy ra (α + α)2 = 1. Mặt khác, từ
α2 − 2α + 2


2

= |α|4 − (2 |α|2 + 4)(α + α) + 2(α + α)2 + 4
= 7 − 6(α + α)

và |α2 − 2α + 2| = 1 ta có α + α = 1.
kiện α.α = |α|2 = 1 ta suy ra
√ Kết hợp với sự √
3i
3i
1
1
và α = +
và 2 nghiệm này phải có
P (x) có đúng 2 nghiệm là α = +
2
2
2
2
bội bằng nhau. Vậy,
P (x) = (x − α)n .(x − α)n = (x2 − x + 1)n .
Ví dụ 3. Tìm tất cả các đa thức P (x) ∈ R[x] khác hằng sao cho
P (x)P (x + 1) = P (x2 ), ∀x ∈ R.
Giải. Gọi α là một nghiệm bất kì (thực hoặc phức) của P (x), thì dễ dàng thấy
n
rằng α2 , α4 , . . . , α2 , . . . cũng là nghiệm của P (x). Do P (x) không phải đa thức
hằng nên tồn tại 2 số nguyên dương m, n(m < n) sao cho
n

m


m

α2 = α2 ⇐⇒ α2
Do đó, hoặc α = 0 hoặc |α| = 1.

n −2m

α2

− 1 = 0.


6
i) Nếu α = 0, thì từ phương trình trong đề bài, ta cho x = 0, được P (1) = 0, suy
ra x = 1 cũng là nghiệm.
ii) Nếu |α| = 1, thì từ giả thiết, bằng cách cho x = α − 1, ta thấy (α − 1)2 cũng
là nghiệm, suy ra hoặc |(α − 1)2 | = 0 hoặc |(α − 1)2 | = 1. Nếu |(α − 1)2 | = 0,
thì α = 1, còn nếu |(α − 1)2 | = 1, thì
1 = (α − 1)2 = |α − 1|2 = (α − 1)(α − 1) = 2 − (α + α),
tức là α + α = 1. Bây giờ từ giả thiết ta cho x = α2 − 1, thì được nghiệm là
(α2 − 1)2 , suy ra |α2 − 1| = 0 hoặc |α2 − 1| = 1.
• Nếu |α2 − 1| = 0, thì α2 = 1 hay α = ±i. Trường hợp này α + α = 0.
Mâu thuẫn với α + α = 1.
• Nếu |α2 − 1| = 1, thì
1 = α2 − 1

2

= α2 − 1


α2 − 1 = 4 − (α + α)2 = 3.

Điều này là vô lí !
Vậy trong mọi trường hợp, P (x) chỉ có nghiệm là 0 và 1. Do đó, P (x) =
axm (x − 1)n , (a = 0; m, n ∈ N∗ ). Thay vào phương trình trong đề bài ta được
a = 1 và m = n. Vậy P (x) = xn (x − 1)n .

Ví dụ 4 (VMO, 2015). Cho (fn (x)) là dãy các đa thức xác định bởi
f0 (x) = 2, f1 (x) = 3x, fn (x) = 3xfn−1 (x) + (1 − x − 2x2 )fn−2 (x), ∀n ≥ 2.
Tìm tất cả các số nguyên dương n sao cho fn (x) chia hết cho x3 − x2 + x.
Giải. Vì phương trình x3 − x2 + x = 0 có 3 nghiệm là x = 0 và x =

fn (x) chia hết cho x3 − x2 + x khi và chỉ khi fn (0) = 0 và fn

1
2

+



3
.i
2

1
2

±




3
i
2

nên

= 0. Từ giả

thiết, ta suy ra fn (0) = fn−2 (0) và
fn


1
3
+
.i
2
2

=


3 3 3
+
.i fn−1
2
2




3
1
3 3 3
+
.i +

.i fn−2
2
2
2
2


3
1
+
.i .
2
2


7
Từ f0 (0) = 2 và f1 (0) = 0 ta suy ra fn (0) = 0 khi và chỉ khi n là số lẻ. Rõ ràng,

là một dãy truy hồi tuyến tính cấp 2 có phương trình đặc trưng là
fn 12 + 23 .i



3
3
3
3
3
3
+
.i t − +
.i = 0.
t2 −
2
2
2
2


3
3
Phương trình này có 2 nghiệm là t1 = +
.i và t2 = 3.i. Từ đây dễ dàng tính
2
2
được


n
√ n
1
3

3 i
fn
+ in .
+
.i = ( 3)
+
2
2
2
2


3
i
π
π
π
π
Viết
+ = cos + i. sin và i = cos + i. sin . Khi đó, fn 21 + 23 .i = 0
2
2
6
6
2
2
nếu và chỉ nếu

cos nπ + cos nπ = 0





6
2
cos
= 0 ⇐⇒
+ cos
+ i sin
+ sin
sin nπ + sin nπ = 0
6
2
6
2
6
2
⇐⇒ n = 3(2k + 1).

Vậy tất cả các số nguyên dương của n để fn (x) chia hết cho x3 − x2 + x là n =
3(2k + 1), k ∈ N.

Ví dụ 5. Cho đa thức f (x) ∈ R[x] sao cho f (x) ≥ 0, ∀x ∈ R. Chứng minh rằng
tồn tại các đa thức g(x), h(x) ∈ R[x] sao cho
f (x) = g 2 (x) + h2 (x), ∀x ∈ R.
Giải. Gọi hệ số cao nhất của f (x) là a. Vì f (x) ≥ 0, ∀x ∈ R nên a > 0. Hơn nữa,
nếu f (x) có nghiệm thực x = a, thì nghiệm này phải có số bội chẵn. Do đó, ta có
thể giả sử
f (x) = a(x − x1 )2k1 (x − x2 )2k2 . . . (x − xm )2km g(x),


trong đó x1 , x2 , . . . , xm là tất cả các nghiệm thực (nếu có) của f (x) và g(x) là đa thức
monic không có nghiệm thực. Ngoài ra, do f (x) ≥ 0, ∀x ∈ R nên g(x) > 0, ∀x ∈ R.
Thế thì g(x) phải có bậc chẵn. Đặt deg(g(x)) = 2n, thì g(x) có 2n nghiệm phức.
Chú ý là do g(x) ∈ R[x] nên nếu α là một nghiệm phức của g(x), thì α cũng là
nghiệm của g(x) với số bội bằng số bội của α. Gọi α1 , α2 , . . . , αn , α1 , α2 , . . . , αn là
tất cả các nghiệm phức của g(x), thì g(x) phân tích được thành
g(x) = [(x − α1 )(x − α1 )]h1 .[(x − α2 )(x − α2 )]h2 . . . [(x − αn )(x − αn )]hn .


8
Với mỗi j = 1, 2, . . . , n ta viết αj = aj + bj .i, aj , bj ∈ R thì
(x − αj )(x − αj ) = x2 − (αj + αj )x + αj .αj
x2 − 2aj x + a2j + b2j = (x − aj )2 + b2j .
Vì tích của các tổng 2 bình phương lại là tổng 2 bình phương nên g(x) có thể viết
thành
g(x) = u2 (x) + v 2 (x), ∀x ∈ R,

ở đó u(x), v(x) ∈ R[x]. Bây giờ nếu đặt
g(x) =



m

(x − xj ) .u(x) và h(x) =
kj

a.
j=1




m

a.
j=1

(x − xj )kj .v(x),

thì f (x) = g 2 (x) + h2 (x), ∀x ∈ R.
Dưới đây là một số bài tập có thể sử dụng số phức để giải.
Bài 1 (Biên của nghiệm). Cho đa thức P (x) = an xn + an−1 xn−1 + · · · + ax + a0 ∈
Z[x], an = 0. Đặt M = max {|ai | : i = 0, 1, 2, . . . , n − 1} . Chứng minh rằng nếu α
M
là một nghiệm của P (x), thì |α| < 1 +
.
|an |
Bài 2. Tìm tất cả các số nguyên dương n sao cho đa thức f (x) = (x + 1)n + xn + 1
chia hết cho g(x) = x2 + x + 1.
Bài 3. Tìm tất cả các đa thức P (x) ∈ R[x] sao cho P (x)P (x − 3) = P (x2 ), ∀x ∈ R.
Bài 4. Tìm tất cả các đa thức P (x) ∈ R[x] sao cho
P (x)P (x + 1) = P (x2 + x + 1), ∀x ∈ R.
Bài 5. Cho các đa thức P (x), Q(x), R(x), S(x) ∈ R[x] thỏa mãn
P (x5 ) + x.Q(x5 ) + x2 .R(x5 ) = (1 + x + x2 + x3 + x4 )S(x), ∀x ∈ R.
Chứng minh rằng P(x) chia hết cho x − 1.


9

2.2


Chứng minh đa thức bất khả quy

Để chứng minh một đa thức P (x) ∈ Z[x] là bất khả quy trên Z ta hãy sử dụng
phương pháp phản chứng, tức là giả sử P (x) là khả quy. Khi ấy, tồn tại các đa thức
f (x), g(x) ∈ Z[x] có bậc lớn hơn hoặc bằng 1 sao cho P (x) = f (x).g(x), ∀x ∈ R.
Từ đây bằng cách sử dụng các kết quả về đa thức, đặc biệt kết quả về nghiệm của
đa thức, ta tìm được một điều vô lí hay mâu thuẫn nào đó và kết thúc chứng minh.
Sau đây là các ví dụ minh họa.
Trong suốt mục này nếu không nói gì thêm, ta hiểu bất khả quy (khả quy) nghĩa là
bất khả quy (khả quy) trên Z.
Ví dụ 1 (JMO,1999). Cho n là số nguyên dương. Chứng minh rằng đa thức P (x) =
(x2 + 12 )(x2 + 22 ) . . . (x2 + n2 ) + 1 là bất khả quy.
Giải. Giả sử f (x) khả quy trên Z, tức là f (x) = g(x).h(x), với g(x), h(x) ∈ Z[x]
và có bậc lớn hơn hoặc bằng 1. Thế thì
g(k.i).h(k.i) = f (k.i) = 1, ∀k ∈ Z, 1 ≤ |k| ≤ n.
Vì g(x), h(x) ∈ Z[x] nên g(k.i) và h(k.i) có dạng a + b.i với a, b ∈ Z, suy ra |g(k.i)|2
và |h(k.i)|2 là các số tự nhiên. Do đó, từ |g(k.i)|2 .|h(k.i)|2 = 1 suy ra |g(k.i)| = 1 và
h(k.i) =

1
= g(k.i) = g(k.i) = g(−k.i).
g(k.i)

Điều này dẫn tới đa thức h(x) − g(−x) có 2n nghiệm. Hơn nữa, đa thức này có
bậc nhỏ hơn 2n nên h(x) = g(−x), ∀x. Như thế, f (x) = g(x).g(−x) và do đó,
f (0) = [g(0)]2 , tức là (n!)2 + 1 = [g(0)]2 . Dễ thấy rằng điều này là không thể. Vậy
điều giả sử là sai hay f (x) là đa thức bất khả quy.
Ví dụ 2 (IMO,1993). Chứng minh rằng đa thức f (x) = xn + 5xn−1 + 3 bất khả
quy trên Z.

Giải. Giả sử f (x) là một đa thức khả quy, tức là tồn tại g(x), h(x) ∈ Z[x] có bậc
lớn hơn hoặc bằng 1 sao cho f (x) = g(x).h(x). Vì g(−5).h(−5) = f (−5) = 3 nên
hoặc |g(−5)| = 1 hoặc |h(−5)| = 1. Giả sử |g(−5)| = 1. Đặt k = deg(g(x)) và gọi
α1 , α2 , . . . , αk là các nghiệm phức của g(x), thì
g(x) = (x − α1 )(x − α2 ) . . . (x − αk ).


10
Từ đây suy ra |(α1 + 5)(α2 + 5) . . . (αk + 5)| = |g(−5)| = 1. Vì f (αi ) = 0 nên
αin−1 (αi + 5) = −3, ∀i = 1, k. Do đó,
|α1 .α2 . . . αk |n−1 .|(α1 + 5)(α2 + 5) . . . (αk + 5)| = 3k .
Để ý là |α1 .α2 . . . αk | = |g(0)| và g(0).h(0) = f (0) = 3 nên |g(0)| = 1 hoặc |g(0)| = 3.
Vì 3k > 1 nên |g(0)| > 1, suy ra |g(0)| = 3. Khi ấy, 3n−1 = 3k hay k = n − 1 =⇒
deg(h(x)) = 1, tức là f (x) phải có nghiệm nguyên x = a. Điều đó là không thể xảy
ra. Vậy f (x) là đa thức bất khả quy.
Ví dụ 3. Cho a, m, n là các số nguyên dương và số nguyên tố p < a − 1. Chứng
minh rằng đa thức P (x) = xm (x − a)n + p bất khả quy.
Giải. Giả sử P (x) khả quy. Thế thì tồn tại các đa thức f (x), g(x) ∈ Z[x] có bậc
lớn hơn hoặc bằng 1 sao cho P (x) = f (x)g(x), ∀x ∈ R. Vì p = P (0) = f (0)g(0) nên
có thể giả sử f (0) = 1. Đặt k = deg(f (x)) và gọi x1 , x2 , . . . , xk là các nghiệm phức
của f (x). Do P (x) là monic nên có thể giả sử f (x) cũng monic. Khi ấy,
f (x) = (x − x1 )(x − x2 ) . . . (x − xk ).
Từ đây suy ra |x1 x2 . . . xk | = |f (0)| = 1. Vì xi , (1 ≤ i ≤ k) cũng là nghiệm của
n
P (x) nên xm
i (xi − a) = −p. Do đó,
n
m
n
m

n
k
|f (a)|n = |(a−x1 )(a−x2 )...(a−xk )|n = |xm
1 (x1 −a) |.|x2 (x2 −a) |...|xk (xk −a) | = p .

Từ đây suy ra |f (a)| = p và k = n. Theo đó,
f (a) − f (0) ∈ {p − 1, p + 1, −p + 1, −p − 1} .
Mà a | f (a) − f (0) nên a | p − 1 hoặc a | p + 1. Nhưng cả hai tình huống này đều
không xảy ra, vì 0 < p − 1 < p + 1 < a. Vậy điều giả sử là sai hay P (x) phải là một
đa thức bất khả quy.
Ví dụ 4. Cho p là số nguyên tố và an an−1 ...a1 a0 , an > 1 là biểu diễn của p trong
hệ thập phân. Chứng minh rằng đa thức P (x) = an xn + an−1 xn−1 + · · · + a1 x + a0
bất khả quy.
Giải. Giả sử ngược lại P (x) là đa thức khả quy. Khi đó tồn tại Q(x), H(x) ∈ Z[x]
có bậc lớn hơn hoặc bằng 1 sao cho P (x) = Q(x).H(x). Vì
P (10) = an .10n + an−1 .10n−1 + · · · + a1 .10 + a0 = p


11
nên hoặc |Q(10)| = 1 hoặc |H(10)| = 1. Giả sử |Q(10)| = 1 và gọi x1 , x2 , ..., xk ,
với k = deg(Q(x)) là tất cả các nghiệm phức của Q(x), thì Q(x) = a(x − x1 )(x −
x2 )...(x − xk ), a ∈ Z, a = 0. Đặt M = max {a0 , a1 , ..., an−1 } , thì dễ dàng chứng
M
9
minh được |xi | ≤ 1 +
≤ 1 + < 6. Từ đó suy ra
an
2
1 = |Q(10)| = |a|.|10 − x1 |.|10 − x2 |...|10 − xk | > 1.
Điều vô lí này chứng tỏ P (x) là bất khả quy.

Ví dụ 5. Cho n ∈ N∗ . Chứng minh rằng đa thức f (x) = (x2 − 7x + 6)2n + 13 bất
khả quy.
Giải. Giả sử f (x) là đa thức khả quy, tức là tồn tại g(x), h(x) ∈ Z[x] có bậc lớn
hơn hoặc bằng 1 sao cho f (x) = g(x).h(x). Gọi xi , i = 1, k là tất cả các nghiệm
phức của g(x), trong đó k = deg(g(x)). Khi ấy,
g(x) = (x − x1 )(x − x2 ) . . . (x − xk ).
Từ đây suy ra

Do đó,

|g(1)| = |(x1 − 1)(x2 − 1) . . . (xk − 1)| ,
|g(6)| = |(x1 − 6)(x2 − 6) . . . (xk − 6)| .
(x21 − 7x1 + 6)(x22 − 7x2 − 6) . . . (x2k − 7xk + 6)

2n

= g 2n (1).g 2n (6).

Vì xi , i = 1, k cũng là nghiệm của g(x) nên (x2i − 7xi + 6)2n = −13. Thành thử,
(x21 − 7x1 + 6)(x22 − 7x2 − 6) . . . (x2k − 7xk + 6)

2n

= 13k .

Như thế, g 2n (1).g 2n (6) = 13k . Vì 13 là số nguyên tố nên k = 2n, suy ra |g(1)| . |g(6)| =
13. Nhưng 5 | g(6) − g(1) nên g(6) = g(1). Suy ra 13 = g 2 (1). Vô lí !
Chú ý. Ta có vài chú ý sau về bài toán này.
i) Bài toán trên là một kết quả mạnh hơn bài toán trong kì thi VMO năm 2014:
”Cho đa thức P (x) = (x2 − 7x + 6)2n + 13, n ∈ N∗ . Chứng minh rằng P (x)

không thể phân tích thành tích của n+ 1 đa thức khác hằng với hệ số nguyên.”


12
ii) Nếu thay 13 bởi một số nguyên tố p sao cho p ≡ ±1 (mod 5), thì kết quả vẫn
không thay đổi.
Ví dụ 6. Cho n là các số nguyên dương và p là một số nguyên tố. Chứng minh
rằng đa thức f (x) = xp − x + pn bất khả quy.
Giải. Giả sử f (x) khả quy, tức là tồn tại các đa thức g(x), h(x) ∈ Z[x] có bậc lớn
hơn hoặc bằng 1 sao cho f (x) = g(x).h(x), ∀x ∈ R. Từ pn = f (0) = g(0).h(0) suy
ra |g(0)| = pa và |h(0)| = pb . Không mất tính tổng quát, ta có thể coi 0 ≤ a ≤ b.
Viết
g(x) = xk + ak−1 xk−1 + · · · + a1 x + a0

và h(x) = xm + bm−1 xm−1 + · · · + b1 x + b0 ,

trong đó a0 = ±pa và b0 = ±pb . Bằng cách đồng nhất hệ số, ta có
−1 = a1 b0 + a0 b1 = pa (±b1 ± a1 pb−a ).
Suy ra a = 0 và do đó, |g(0)| = 1. Gọi x1 , x2 , . . . , xk là các nghiệm của g(x), thì
g(x) = (x − x1 )(x − x2 ) . . . (x − xk ) =⇒ |x1 x2 . . . xk | = |g(0)| = 1.
Từ đây ta thấy phải tồn tại i (1 ≤ i ≤ k) sao cho |xi | ≤ 1. Do xi cũng là nghiệm
của f (x) nên
pn = |xi − xpi | ≤ |xi | + |xi |p ≤ 2.
Điều này chỉ có thể xảy ra nếu p = 2 và n = 1. Khi đó, f (x) = x2 − x + 2. Nhưng
đây không phải là một đa thức khả quy. Vậy điều giả sử là sai hay f (x) phải là một
đa thức bất khả quy.

Ví dụ 7. Cho p là số nguyên tố và a là số nguyên không chia hết cho p. Chứng
minh rằng P (x) = xp − x + a bất khả quy.
Giải. Giả sử P (x) khả quy. Khi ấy tồn tại Q(x), H(x) ∈ Z[x] có bậc lớn hơn hoặc

bằng 1 sao cho P (x) = Q(x).H(x). Đặt k = deg(Q(x)) và gọi x1 , x2 , . . . , xk là tất
cả các nghiệm phức của Q(x). Ta có xpi = xi − a, ∀i = 1, k. Do đó
xp1 + xp2 + · · · + xpk = x1 + x2 + · · · + xk − k.a
Vì Q(x) ∈ Z[x] nên theo định lí Vieta, thì các đa thức đối xứng cơ bản của
x1 , x2 , . . . , xk đều là số nguyên. Do đó tất cả các đa thức đối xứng đối với các biến


13
x1 , x2 , . . . , xk đều là số nguyên. Nói riêng, x1 +x2 +· · ·+xk ∈ Z và xp1 +xp2 +· · ·+xpk ∈ Z.
Theo định lí nhỏ Fermat, ta có
(x1 + x2 + · · · + xk )p ≡ x1 + x2 + · · · + xk

(mod p).

Mặt khác, ta lại có
xp1 + xp2 + · · · + xpk = (x1 + x2 + · · · + xk )p −

0≤i1 ,i2 ,...,ik i1 +i2 +...+ik =p

p!
xi11 xi22 ...xikk .
i1 !i2 !...ik !

p!
luôn chia hết cho p. Do đó, ta suy ra k.a chia
i1 !i2 !...ik !
hết cho p. Điều này không thể xảy ra vì 1 ≤ k < p và a không chia hết cho p.

Vì 0 ≤ i1 , i2 , ..., ik < p nên


Ví dụ 8. Cho đa thức P (x) = an xn + an−1 xn−1 + · · · + a1 x + a0 ∈ Z[x] thỏa mãn
|a0 | là số nguyên tố và |a0 | > |a1 | + |a2 | + · · · + |an |. Chứng minh rằng P (x) là bất
khả quy trên Z.
Giải. Giả sử P (x) khả quy. Khi ấy tồn tại Q(x), H(x) ∈ Z[x] có bậc lớn hơn hay
bằng 1 sao cho P (x) = Q(x).H(x). Vì |P (0)| = |a0 | là số nguyên tố nên |Q(0)| = 1
hoặc |H(0)| = 1. Giả sử |Q(0)| = 1. Gọi x1 , x2 , ..., xk là tất cả các nghiệm phức
của Q(x), k = deg(Q(x)), thì Q(x) = a.(x − x1 )(x − x2 ) . . . (x − xk ), với a | an . Vì
1
≤ 1. Do đó, tồn tại i, 1 ≤ i ≤ k sao cho |xi | ≤ 1.
|Q(0)| = 1 nên |x1 |.|x2 |...|xk | =
|a|
Rõ ràng, xi cũng là nghiệm của P (x) nên P (xi ) = 0 hay
a0 = −xi a1 + a2 xi + · · · + an xin−1 .
Từ đó suy ra
|a0 | = |xi | . a1 + a2 xi + · · · + an xin−1 ≤ |a1 | + |a2 | + · · · + |an | .
Điều này mâu thuẫn với giả thiết.
Ví dụ 9. Cho p là một số nguyên tố, k, h ∈ N∗ sao cho

rằng f (x) = k.xp − h là đa thức bất khả quy.

p

h

/ Q. Chứng minh
k

Giải. Ta chỉ cần xét (h, k) = 1. Giả sử f (x) = g(x).h(x), với g(x), h(x) ∈ Z[x] và
có bậc lớn hơn hoặc bằng 1. Đặt m = deg(g(x)), 1 ≤ m < pvà gọi ε1 , ε2 , . . . , εm



14
là các nghiệm phức của g(x). Khi đó, εi , ∀i = 1, m là các căn bậc p của

g(x) = a(x − ε1 )(x − ε2 ) . . . (x − εm ), với a là ước dương của k. Ta có

h

k

h = |f (0)| = |g(0)|.|h(0)| = ab|ε1 |.|ε2| . . . |εm |,
trong đó b = |h(0)| là ước dương của h. Từ đó suy ra
p

p

p

h = a .b

.|εp1 |.|εp2| . . . |εpm |

p

p

= a .b

h

k

m

.

k = c.au
thì aum .bv(p−m) .cm .dp−m = ap .bp . Vì
h = d.bv
a, b, c, d đôi một nguyên tố cùng nhau nên

 p
m=1
a = aum






b = 1
bp = bv(p−m)
h=1
=⇒
=⇒


c=1
k = ap
cm = 1






 p−m
d
=1
d=1
Suy ra hp−m .k m = ap .bp . Đặt

Thành thử,

p

1
h
= ∈ Q. Mâu thuẫn !
k
a

Ví dụ 10. Cho p là số nguyên tố dạng 4k + 3. Chứng minh rằng đa thức f (x) =
(x2 + 1)n + p là bất khả quy.
Giải. Giả sử ngược lại f (x) = g(x)h(x), trong đó g(x), h(x) ∈ Z[x] và có bậc lớn
hơn hoặc bằng 1. Ta có
p = h(i) = g(i)h(i) =⇒ p2 = |g(i)|2.|h(i)|2

(1)

Vì các số |g(i)|2 và |h(i)|2 đều có dạng a2 + b2 với a, b ∈ Z và p là số nguyên tố dạng

4k + 3 nên không thể xảy ra trường hợp |g(i)|2 = |h(i)|2 = p. Do đó, ta có thể giả
sử |g(i)| = 1. Đặt k = deg(g(x)) và gọi x1 , x2 , ..., xk là nghiệm của g(x), thì từ f (x)
là monic ta suy ra g(x) cũng vậy. Hơn nữa, ta có
g(x) = (x − x1 )(x − x2 ) . . . (x − xk ).
Khi đó,
1 = |g(i)|2 = g(i)g(i) = g(i)g(−i) = (x21 + 1)(x22 + 1) . . . (x2k + 1).


15
Để ý là xj , j = 1, 2, . . . , k cũng là nghiệm của f (x) nên x2j + 1

1 = (x21 + 1)n (x22 + 1)n . . . (x2k + 1)n = (−p)k .

n

= −p. Do đó, ta

Đây là điều vô lí !
Ví dụ 11 (Việt Nam TST, 2013). Tìm tất cả các số nguyên n > 1 và số nguyên
tố p sao cho đa thức f (x) = xn − px + p2 khả quy trên Z.
Giải. Giả sử f (x) = g(x)h(x), trong đó g(x), h(x) ∈ Z[x] có bậc lần lượt là
k, m ≥ 1 (k + m = n). Từ p2 = f (0) = g(0).h(0) suy ra |g(0)| ∈ {1; p; p2 } . Nhưng
thực tế ta chỉ cần quan tâm đến các trường hợp |g(0)| = 1 hoặc |g(0)| = p. Gọi
x1 , x2 , . . . , xk là các nghiệm phức của g(x), thì
g(x) = (x − x1 )(x − x2 ) . . . (x − xk ).
• Nếu |g(0)| = 1, thì |x1 x2 . . . xk | = 1. Khi đó, do xni = p(xi − p), ∀i = 1, k nên
1 = |x1 x2 . . . xk |n = pk . |(x1 − p)(x2 − p) . . . (xk − p)| = pk . |g(p)| .
Đây là điều vô lí !
• Nếu g(0) = h(0) = ±p, thì từ |x1 x2 . . . xk | = |g(0)| = p ta suy ra
pn = |x1 x2 . . . xk |n = pk . |(x1 − p)(x2 − p) . . . (xk − p)| = pk . |g(p)| .

Thành thử, |g(p)| = pm . Khi đó, ta có |h(p)| = pk . Do |g(0)| = |h(0)| = p nên
g(−p) và h(−p) đều chia hết cho p. Mà
g(−p).h(−p) = f (−p) = p2 2 + (−p)n−2
nên g(−p) = a.p và h(−p) = b.p, trong đó a, b ∈ Z sao cho ab = 2 + (−p)n−2 .
Ta có
g(p) + g(−p) = 2g(0) + A.p2
g(p) + g(−p) = a.p + pm



h(p) + h(−p) = 2g(0) + B.p2
h(p) + h(−p) = b.p + pk

với A, B ∈ Z. Từ đây suy ra nếu m ≥ 2 và k ≥ 2, thì

a.p − 2g(0) ... p2
a ≡ −2 (mod p)
a ≡ 2 (mod p)
hoặc
=⇒
b.p − 2g(0) ... p2
b ≡ −2 (mod p)
b ≡ 2 (mod p)


16
Cả hai trường hợp ta đều có ab ≡ 4 (mod p), suy ra 2 + (−p)n−2 ≡ 4 (mod p)
.
.
hay p = 2. D đó, a và b là các số chẵn nên ab .. 4 hay 2 + (−2)n−2 .. 4. Nhưng

điều này không xảy ra vì n = m + k ≥ 4. Như vậy bắt buộc phải có k = 1
hoặc m = 1, tức là f (x) phải có nghiệm nguyên x0 . Từ xn0 = p(x0 − p) suy ra
.
x0 .. p hay x0 = t.p. Do đó, tn pn−2 = t − 1. Phương trình này chỉ có nghiệm
t = −1, p = 2, n = 3.

Thử lại ta thấy n = 3, p = 2 thỏa mãn đề bài và đó là tất cả các giá trị cần tìm.
Ví dụ 12. Cho p là số nguyên tố có dạng 4k + 3 và a, b là các số nguyên sao cho
min {vp (a), vp (b − 1)} = 1. Chứng minh rằng đa thức f (x) = x2p + a.x + b là bất
khả quy.
Giải. Giả sử f (x) khả quy, tức là f (x) = g(x).h(x) với g(x), h(x) ∈ Z[x] và có bậc
lớn hơn hoặc bằng 1. Ta có
|f (i)|2 = |b − 1 + a.i|2 = a2 + (b − 1)2 .
Đặt

b − 1 = c.pu
a = d.pv

(với c, d ∈ Z; (c, p) = (d, p) = 1 và u, v ∈ N∗ ; min {u, v} = 1).

Khi ấy
a2 + (b − 1)2 = p2 (c2 + d2 .p2v−2 )
a2 + (b − 1)2 = p2 (c2 .p2u−2 + d2 )

Vì (c, p) = (d, p) = 1 và p là số nguyên tố có dạng 4k+3 nên c2 +d2 .p2v−2 và c2 .p2u−2 +
d2 đều không chia hết cho p. Do đó, vp |f (i)|2 = 2. Rõ ràng là |g(i)|2 , |h(i)|2 ∈ N∗ .
Do đó, từ |g(i)|2 . |h(i)|2 = |f (i)|2 và p là số nguyên tố có dạng 4k + 3 nên chỉ
có thể xảy ra hoặc vp |g(i)|2 = 2 hoặc vp |h(i)|2 = 2. Giả sử vp |g(i)|2 = 2 thì
vp |h(i)|2 = 0. Đặt k = deg(h(x)) và gọi x1 , x2 , ..., xk là tất cả các nghiệm phức của
h(x) thì dễ thấy h(x) là monic và h(x) = kj=1 (x − xj ). Ta có h(i) = kj=1 (i − xj ),

suy ra
p

k

k
2

j=1

(x2j + 1) = |h(i)| =⇒

(x2j + 1)
j=1

= |h(i)|2p .

1 p−1 l 2l a
b−1
. Từ đó suy ra
Cp x − x −
p l=1
p
p
= p.g(xj ), ∀j = 1, k. Như thế, ta có

Để ý là f (x) = (x2 + 1) − p.g(x) với g(x) =
p

x2j + 1


p

p

k

(x2j
j=1

+ 1)

k

= |h(i)|

2p

=p

k

g(xj ) ,
j=1

(∗).


17
Vì h(x) ∈ Z[x] nên theo định lí Vieta thì các đa thức đối xứng cơ bản của x1 , x2 , ..., xk

là các số nguyên. Do đó, tất cả các đa thức đối xứng với hệ số nguyên của x1 , x2 , ..., xk
k
cũng là số nguyên. Dễ thấy g(x) ∈ Z[x] nên
j=1 g(xj ) là số nguyên. Thành thử
từ (∗) suy ra |h(i)|2p chia hết cho p. Điều này là vô lí !

Ví dụ 13. Chứng minh rằng đa thức f (x) = (x2 + x)2 + 1 bất khả quy với mọi
n ∈ N∗
n

Giải. Giả sử f (x) = g(x).h(x), trong đó g(x), h(x) ∈ Z[x] có bậc lớn hơn hoặc
bằng 1.
√ có thể giả thiết g(x)
√ và h(x) cũng vậy. Đặt ε =
√ Vì f (x) là monic nên
a + 3b.i
c + 3d.i
−1 + 3i
, thì g(ε) =
và h(ε) =
, trong đó a, b, c, d ∈ Z và
2
2k
2k
(a, b, 2) = (c, d, 2) = 1. Từ
2 = f (ε) = g(ε).h(ε) =⇒ a2 + 3b2

c2 + 3d2 = 4k+m+1 .

Do phương trình x2 + 3y 2 = 2 không có nghiệm nguyên nên

a2 + 3b2 = 4u
c2 + 3d2 = 4v

(u + v = k + m + 1).

Ta có thể giả sử u ≤ v. Khi ấy xảy ra 2 trường hợp sau
• Nếu u = 0, thì a2 = 1, b = 0. Khi ấy v > 0 và do đó c, d phải có cùng lẻ, suy
ra
c2 + 3d2 ≡ 4 (mod 8) =⇒ v = 1.
Từ đây ta có k + m = 0 ⇐⇒ k = m = 0 hay |g(ε)| = 1 và |h(ε)| = 2.

• Nếu u > 0, thì v > 0. Lập luận tương tự như trên, ta được u = v = 1 hay
k + m = 1. Do đó hoặc k = 1, m = 0 hoặc k = 0, m = 1. Thành thử hoặc
|g(ε)| = 1 và |h(ε)| = 2 hoặc là |g(ε)| = 2 và |h(ε)| = 1.
Tóm lại, không mất tính tổng quát, ta có thể giả sử |g(ε)| = 1. Đặt t = deg(g(x))
và gọi x1 , x2 , ..., xt là các nghiệm phức của g(x), thì
g(x) = (x − x1 )(x − x2 ) . . . (x − xt ) =⇒ |g(ε)| = |(x1 − ε)(x2 − ε)...(xt − ε)| .
Từ đây suy ra
(x21 + x1 + 1)(x22 + x2 + 1) . . . (x2t + xt + 1) = |g(ε)|2 = 1.


18
Đặt q(x) =

2n

C2kn (−1)k (x2 + x + 1)k−1 , thì do

k=1
n


(x2i + xi + 1)q(xi ) + 2 = (x2i + xi )2 + 1 = 0, ∀i = 1, t
nên ta có
|g(ε)|2 .q(x1 )q(x2 ) . . . q(xt ) = (−2)t .

Vì q(x) ∈ Z[x] nên các biểu thức đối xứng cơ bản của x1 , x2 , ..., xt là các số
nguyên, suy ra các biểu thức đối xứng đối với x1 , x2 , ..., xt cũng vậy. Nói riêng,
.
q(x1 )q(x2 ) . . . q(xt ) là một số nguyên. Để ý là C kn .. 2, ∀k = 1, 2n nên ta đặt
n −1

q(x) = (x2 + x + 1)2

+ 2.k(x). Do đó,
n −1)

q(x1 )q(x2 ) . . . q(xt ) = |g(ε)|2(2

2

+ 2.k(x1 , x2 , ..., xt ) = 1 + 2.k(x1 , x2 , ..., xt ),

với k(x1 , x2 , ..., xt ) là biểu thức đối xứng đối với x1 , x2 , ..., xt . Thành thử, k(x1 , x2 , ..., xt ) ∈
Z và q(x1 )q(x2 ) . . . q(xt ) là một số lẻ. Do đó, (−2)t là số nguyên lẻ. Đây là điều vô
lí, vì t ≥ 1.

2.3

Chứng minh một số tiêu chuẩn bất khả quy bằng số
phức


Trong mục này chúng tôi sẽ sử dụng số phức để chứng minh một số kết quả quan
trọng về đa thức bất khả quy. Kết quả đầu tiên là tiêu chuẩn bất khả quy của
Eisenstein. Đây là kết quả rất đẹp và quan trọng. Nhưng trong các tài liệu mà
chúng tôi biết, thì chưa có một tài liệu nào chứng minh bằng số phức. Sau khi cố
gắng để tìm một lời giải bằng số phức cho bài toán về đa thức bất khả quy trong
kì thi chọn đội tuyển dự thi IMO năm 2013, chúng tôi đã mạnh dạn tìm một chứng
minh bằng số phức cho tiêu chuẩn này và sau một thời gian suy nghĩ, chúng tôi đã
tìm được một chứng minh đẹp đẽ và khá là đơn giản như trình bày dưới đây.
Định lý 1 (Tiêu chuẩn Eisenstein). Cho P (x) = an xn + an−1 xn−1 + · · · + a1 x + a0 ∈
Z[x]. Nếu tồn tại một số nguyên tố p sao cho 3 điều kiện sau đồng thời thỏa mãn
i) a0 , a1 , . . . , an−1 chia hết cho p;
ii) an không chia hết cho p;
iii) a0 không chia hết cho p2 ,


19
thì P (x) là đa thức bất khả quy trên Z.
Chứng minh. Giả sử P (x) không phải là đa thức bất khả quy. Khi ấy tồn tại các đa
thức f (x), g(x) ∈ Z[x] có bậc lớn hơn hoặc bằng 1 sao cho P (x) = f (x)g(x), ∀x ∈
R. Vì a0 = P (0) = f (0)g(0) và vp (a0 ) = 1 nên ta có thể giả sử f (0) = b, với
b | a0 , (b, p) = 1. Đặt k = deg(f (x)) và gọi x1 , x2 , . . . , xk là các nghiệm (thực hoặc
phức) của f (x), thì
f (x) = c(x − x1 )(x − x2 ) . . . (x − xk ),

trong đó c là một số nguyên và c | an . Ta có

b = f (0) = c.(−1)k x1 x2 . . . xk =⇒ x1 x2 . . . xk =

(−1)k b

.
c

Do x1 , x2 , . . . , xk cũng là nghiệm của P (x) nên
an xni = −an−1 xin−1 + · · · + a1 xi + a0 , ∀i = 1, k.

Nhân k đẳng thức này theo vế, ta thu được
k
k

n

|an | .|x1 x2 . . . xk | =

i=1

(an−1 xin−1 + · · · + a1 xi + a0 ) ,

hay
b
c

akn .

k

n

=
i=1


(an−1 xin−1 + · · · + a1 xi + a0 )

(1).

Theo định lí Vieta, ta thấy tất cả các đa thức cơ bản của x1 , x2 , . . . , xk đều là các số
hữu tỉ có mẫu chung là an . Vì thế tất cả các đa thức đối xứng với hệ số nguyên của
các biến x1 , x2 , . . . , xk cũng là số hữu tỉ mà mẫu số là lũy thừa nào đó của an . Rõ
k
n−1
+ · · · + a1 xi + a0 ) là một đa thức đối xứng với hệ số nguyên
ràng,
i=1 (an−1 xi
của các biến x1 , x2 , . . . , xk nên
k

i=1

(an−1 xin−1 + · · · + a1 xi + a0 ) =

M
,
|an |m

(2)

.
.
trong đó M, m ∈ N. Hơn nữa, do a0 , a1 , . . . , an−1 .. p nên M .. p. Thay (2) vào (1)
được

n
M
b
=
akn .
⇐⇒ am+k
.bn = M|c|n .
n
c
|an |m


20
.
.bn .. p. Vô lí ! Vậy điều giả ban đầu là sai hay P (x) là một đa
Từ đây suy ra am+k
n
thức bất khả quy.
Chú ý. Nhờ vào công cụ số phức ta có thể chứng minh được tiêu chuẩn Eisenstein
mở rộng.
” Cho đa thức P (x) = an xn + an−1 xn−1 + · · ·+ a1 x+ a0 ∈ Z[x]. Nếu tồn tại số nguyên
tố p và số nguyên 0 ≤ k ≤ n sao cho 3 điều kiện sau đây đồng thời thỏa mãn
i) a0 , a1 , ..., ak−1 chia hết cho p;
ii) ak không chia hết cho p;
iii) a0 không chia hết cho p2
thì trong phân tích P (x) thành nhân tử có bậc lớn hơn hoặc bằng k.”
Định lý 2 (Tiêu chuẩn Cohn). Cho p là một số nguyên tố và an an−1 . . . a1 a0 là biểu
diễn của p trong hệ cơ số b (b ≥ 2). Khi đó P (x) = an xn + · · · + a1 x + a0 là một đa
thức bất khả quy.
Chứng minh. Để chứng minh tiêu chuẩn này ta cần 2 bổ đề sau

Bổ đề 1. Cho đa thức f (x) = an xn +· · ·+a1 x+a0 ∈ Z[x] thỏa mãn an ≥ 1, an−1 ≥ 0.
Đặt M = max |ai |, i = 1, n − 2 . Khi đó nếu α là một nghiệm của f (x), thì hoặc

1 + 1 + 4M
Re(α) ≤ 0 hoặc |α| <
.
2
Chứng minh.
√ Giả sử ngược lại rằng f (x) có một nghiệm α sao cho Re(α) > 0 và
1 + 1 + 4M
. Vì Re(α) = Re(α) > 0 nên
|α| ≥
2
f (α)
an−1 an−2
an−1
|an−2 |
a0
|a0 |

= an +
+ 2 + · · · + n ≥ an +
+···+
n
2
α
α
α
α
α

|α|
|α|n
1
1
an−1
−M
+···+
≥ Re an +
2
α
|α|
|α|n
1
1 − |α|n−1
an−1
= an +
.Re(α) − M.
|α|2
|α|2 − |α|
>1−

M
|α|2 − |α| − M
=
≥ 0.
|α|2 − |α|
|α|2 − |α|

Điều này vô lí, vì α là nghiệm của f (x).



21
Bổ đề 2. Nếu đa thức f (x) = xn + an−1 xn−1 + · · · + a1 x + a0 có các hệ số ai ∈
3
{0; 1} , ∀i = 1, n − 1, thì mọi nghiệm α của nó đều có Re(α) < .
2
Chứng minh. Với n = 1 và n = 2, ta có thể kiểm tra trực tiếp. Ta xét n ≥ 3 và giả
π
3
sử ngược lại f (x) có một nghiệm là α với Re(α) ≥ . Khi đó, arg(α) ≤ , vì nếu
2
4
π
arg(α) > , thì
4


1+ 5
3
|α| > 2.Re(α) ≥ √ >
.
2
2

π
1+ 5
. Vì arg(α) ≤ nên arg(α2 ) ≥
Điều này là vô lí, vì theo bổ đề trên thì |α| <
2
4

0. Mà n ≥ 3 nên ta có đánh giá
f (α)
an−1 an−2
1
1
≥ 1+
+ 2 −
+···+
n
3
α
α
α
|α|
|α|n
1
an−1 an−2
+ 2 −
> 1+
2
α
α
|α| (|α| − 1)
1
an−1 an−2
+ 2 −
≥ Re 1 +
2
α
α

|α| (|α| − 1)
an−1
an−2
1
2
=1+
Re(α)
+
Re(α
)

> 0.
|α|2
|α|4
|α|2(|α| − 1)
Vô lí, vì α là một nghiệm của f (x).
Ta hãy trở lại chứng minh tiêu chuẩn Cohn.
Giả sử P (x) khả quy, tức là tồn tại các đa thức f (x), g(x) ∈ Z[x] có bậc lớn hơn
hoặc bằng 1 sao cho P (x) = f (x).g(x), ∀x ∈ R. Vì p = P (b) = f (b).g(b) nên có thể
giả sử |f (b)| = 1. Đặt k = deg(f (x)) và gọi x1 , x2 , . . . , xk là các nghiệm của f (x),
thì
f (x) = c(x − x1 )(x − x2 ) . . . (x − xk ),
với c | an . Xảy ra 2 trường hợp sau

i) Nếu b ≥ 3, thì với mọi nghiệm α của P (x), ta có


1 + 1 + 4M
1 + 4b − 3
|α| <


,
2
2


22
với M = max {|ai | : i = 0, 1, . . . , n − 2} . Do đó,

1 + 4b − 3
|b − α| ≥ b − |α| > b −
≥ 1.
2
Vì x1 , x2 , . . . , xk cũng là nghiệm của P (x) nên
1 = |f (b)| = |c|.|b − x1 |.|b − x2 | . . . |b − xk | > 1.

Đây là điều vô lí !Do đó, với b ≥ 3, thì P (x) bất khả quy.

ii) Nếu b = 2, thì |f (2)| = 1. Theo bổ đề 2, mọi nghiệm α của P (x) đều thỏa mãn
3
Re(α) < . Khi ấy,
2
|2 − α|2 − |1 − α|2 = (2 − α)(2 − α) − (1 − α)(1 − α) = 3 − 2.Re(α) > 0,

suy ra |2 − α| > |1 − α|. Nhờ sự kiện này, ta có

1 = |f (2)| = |2−x1 |.|2−x2 | . . . |2−xk | > |1−x1 |.|1−x2 | . . . |1−xk | = f (1) ≥ 1.
Điều mâu thuẫn này kết thúc chứng minh của ta.

Định lý 3 (Tiêu chuẩn Perron). Cho đa thức P (x) = xn +an−1 xn−1 +· · ·+a1 x+a0 ∈

Z[x] thỏa mãn a0 = 0 và |an−1 | > 1 + |a0 | + |a1 | + · · · + |an−2 |. Khi ấy, P (x) là một
đa thức bất khả quy.
Chứng minh. Để chứng minh tiêu chuẩn này ta cần tới bổ đề sau
Bổ đề 3. Nếu đa thức P (x) = xn + an−1 xn−1 + · · · + a1 x + a0 ∈ Z[x] thỏa mãn
|an−1 | > 1 + |a0 | + |a1 | + · · · + |an−2 |, thì nó có đúng một nghiệm thỏa mãn |z| > 1
và n − 1 nghiệm còn lại thỏa mãn |z| < 1.
Chứng minh. Ta có cần chứng minh cho trường hợp a0 = 0, vì nếu trái lại, ta chỉ
cần bỏ đi một nhân tử có dạng xk . Gọi α1 , α2 , . . . , αn là các nghiệm của P (x), thì do
|α1 |.|α2 | . . . |αn | = |a0 | ≥ 1 nên tồn tại một nghiệm, chẳng hạn α1 sao cho |α1 | ≥ 1.
Mặt khác nếu P (x) có nghiệm α với |α| = 1, thì
|an−1 | = |an−1 αn−1| ≤ |αn + an−2 αn−2 + · · · + a1 α + a0 |
≤ |α|n + |an−2 |.|α|n−2 + · · · + |a1 |.|α| + |a0 |
≤ 1 + |an−2 | + · · · + |a1 | + |a0 |.


23
Mâu thuẫn với giả thiết. Do đó, |α1 | > 1. Đặt
Q(x) = (x − α2 )(x − α3 ) . . . (x − αn ) = xn−1 + bn−2 xn−2 + · · · + b1 x + b0 ,

thì P (x) = (x − α1 )Q(x). Bằng cách đồng nhất hệ số, ta có

an−1 = bn−2 − α1 ; a0 = −α1 .b0 ; ai = bi−1 − α1 .bi , ∀i = 1, n − 2.

Khi ấy, từ các bất đẳng thức



|an−1 | = |bn−2 − α1 | ≤ |bn−2 | + |α1 |

|an−2 | + · · · + |a1 | + |a0 | = |bn−3 − α1 .bn−2 | + · · · + |b0 − α1 .b1 | + |α1 .b0 |

≥ (|α1 |.|bn−2 | − |bn−3 |) + · · · + (|α1 |.|b1 | − |b0 |) + |α1 |.|b0 |
= (|α1 | − 1)(|bn−2 | + |bn−3 | + · · · + |b1 | + |b0 |) + |bn−2 |
cùng với giả thiết, ta suy ra
|bn−2 | + |bn−3 | + · · · + |b1 | + |b0 | < 1.

Bây giờ nếu αi (2 ≤ i ≤ n) sao cho |αi | ≥ 1, thì do αi là nghiệm của Q(x) nên
1 ≤ |αi |n−1 = |bn−2 αin−2 + bn−3 αin−3 + · · · + b1 αi + b0 |
≤ |bn−2 |.|αi |n−2 + |bn−3 |.|αi |n−3 + · · · + |b1 |.|αi | + |b0 |
≤ |bn−2 | + |bn−3 | + · · · + |b1 | + |b0 | < 1.
Điều vô lí này kết thúc chứng minh bổ đề 3.
Trở lại với chứng minh tiêu chuẩn Perron:
Giả sử P (x) khả quy, suy ra tồn tại f (x), g(x) ∈ Z[x] có bậc lớn hơn hoặc bằng 1
sao cho P (x) = f (x)g(x), ∀x ∈ R. Vì P (x) có đúng một nghiệm thoả mãn |z| > 1
và n − 1 nghiệm còn lại thỏa mãn |z| < 1 nên một trong hai đa thức f (x) hoặc g(x)
chỉ có nghiệm thỏa mãn |z| < 1. Giả sử f (x) là đa thức như vậy, thì f (x) có thể
viết thành

Khi đó,

f (x) = (x − x1 )(x − x2 ) . . . (x − xk ),

k = deg(f (x)).

1 ≤ |f (0)| = |x1 |.|x2 | . . . |xk | < 1.

Điều vô lí này chứng tỏ điều giả sử là sai hay P (x) là một đa thức bất khả quy.


24
Nhờ vào số phức chúng ta có thể chứng minh được một số tiêu chuẩn bất khả quy

khác. Hơn nữa, sự kết hợp giữa số phức và đa thức cũng là một công cụ rất mạnh
để giải các bài toán đếm trong Tổ hợp. Tuy nhiên, những điều này chúng tôi xin để
dành cho chuyên đề khác. Cuối cùng là một số bài tập tự rèn luyện.
Bài 1. Cho đa thức P (x) = xm + xn + 1 với m = deg(P (x)) ≥ 3 và m.n ≡ 2
(mod 3). Chứng minh rằng P (x) là đa thức khả quy.
Bài 2. Chứng minh rằng đa thức f (x) = (x2 − 7x + 6)2 + 1 là bất khả quy.
n

Bài 3. Tìm tất cả các số nguyên tố p để đa thức f (x) = xp + x − p khả quy.

Bài 4. Cho m, n ∈ Z, (n ≥ 2, m ≥ 1) và p là một số nguyên tố. Chứng minh rằng
nếu đa thức f (x) = pm xn + an−1 xn−1 + · · · + a1 x + a0 ∈ Z[x], trong đó a0 .an−1 =
0, an−1 không chia hết cho p sao cho
pm > |a0 | + |a1 | + · · · + |an−1 |,
thì f (x) là một đa thức bất khả quy.
Bài 5. Cho đa thức P (x) = an xn + an−1 xn−1 + · · · + a1 x + a0 ∈ Z[x], an = 0. Đặt
ai
: i = 1, 2, . . . , n − 1 . Chứng minh rằng nếu tồn tại số tự nhiên
M = max
an
m > M + 1 sao cho P (m) là số nguyên tố, thì P (x) là một đa thức bất khả quy.
Bài 6. Cho p, q là 2 số nguyên tố phân biệt và số nguyên n ≥ 3. Tìm tất cả các số
nguyên a sao cho f (x) = xn + axn−1 + pq là một đa thức bất khả quy.
Bài 7. Chứng minh rằng với mọi số nguyên n > 1, đa thức P (x) = xn − x − 1 là
bất khả quy.

Tài liệu
[1] Nguyễn Văn Mậu: Đa thức đại số và phân thức hữu tỉ.
[2] Nguyễn Văn Mậu: Chuyên đề chọn lọc Số phức và Áp dụng.
[3] Tạp chí Toán học và Tuổi trẻ.

[4] Titu Andresscu: Problems from the book.


25
[5] Victor V.Prasolov: Polynomials.
[6] Các
nguồn
tài
liệu
từ


Internet: ;


×